You are on page 1of 30

1

Infinite Series
1.1 Definitions, 1.2 General Properties of Series, 1.3 Comparison Test, 1.4 Integral Test, 1.5 D Alemberts Ratio Test, 1.6 Raabes Test, 1.7 Logarithmic Test, 1.8 Cauchys Root Test, 1.9 Alternating Series, 1.10 Series of Positive and Negative Terms, 1.11 Power Series, 1.12 Uniform Convergence and Weirstrasss M-test.

1.1

DEFINITIONS

I. Sequence. A function whose domain is the set of natural numbers and range a subset of real numbers is called a real sequence or simply a sequence. Symbolically, if u : Natural Numbers real numbers, then u is a sequence. It is denoted as {un}, where un is the n-th term of the sequence or simply u1, u2, u3 1 1 1 ....... Thus 1, , , ...... is an infinite sequence. 2 3 4 II. Monotonic sequence. If in the sequence {un}, (i) un+1 un for all n or, (ii) un+1 un for all n, then the sequence is said to be monotonic increasing (for i) or monotonic decreasing (for ii) sequence.

1 n , {un} is monotonic increasing sequence and if un = 1 + , {un} is monotonic n n +1 decreasing sequence. III. Bounded sequence. If corresponding to the sequence {un} there exists a finite number M such that un M for all n the sequence is said to be bounded above and if un M for all n, the sequence is said to be bounded below. If a sequence is bounded both above and below it is said to be bounded.
Thus, if un = Thus, 1 +

lower bound is 0. IV. Convergent sequence. A sequence {un} is called convergent and is said to have a limit l, if

R S T

1 is bounded above and the upper bound is 2 and n

U V W

R 1 U is bounded below and the SnV TW

Thus,

V. Divergent sequence. The sequence {un} is called divergent if 1

R n U is convergent and the limit is 1. S n + 1V T W

Lt un = l.

VED

C-4\N-MATH\Ch1-1.pm5

Textbook of Engineering Mathematics


n

Lt un = + or .

Thus {n3} is divergent. VI. Oscillatory sequence. If a sequence neither converges nor diverges to + or , then the sequence is said to be oscillatory. Moreover, if it is bounded, it is said to be oscillate finitely or if it is unbounded it is said to be oscillate infinitely. Thus, { 1, 1, 1, 1, 1, 1, ......} oscillates finitely and {( 1)n . n2} oscillates infinitely. Now we state the necessary and sufficient conditions that a sequence {un} may be convergent as follows : (i) Either, the sequence is monotonic and bounded i.e., monotonic increasing and bounded above or monotonic decreasing and bounded below. (ii) Or else, the sequence satisfies Cauchys condition namely, Given and pre-assigned small positive quantity , we can determine a positive integer N, such that | un+p un | < , whenever n N, p being any positive integer. Example 1. Prove that the sequence Let Now un = un+1 un =

n +1 n ,u = 2 1 n+1 ( n + 1) + 1 n +
2

R n U is convergent. S n + 1V T W
2

n2 n + 1 < 0 for all n. [( n + 1) 2 + 1] (n 2 + 1) Then un+1 < un for all n i.e., {un} is monotonic decreasing. n > 0 for all n i.e., {un} is bounded below by 0 Also, un = 2 n +1 Hence the given sequence is convergent 1/n n Lt un = Lt 2 = Lt = 0. Also, 2 n n n +1 n 1 + 1/n Example 2. Show that the sequence {(1 + 1/n)n} is monotonic increasing.
Let Assume that if, if, un = 1 +

F H

1 n

I K

F1 + 1 I H nK FG 1 + 1/n IJ H 1 + 1/(n 1)K

un > un1
n

FG 1 IJ H n 1K F 1 IJ > G1 + H n 1K
> 1+

n 1

VED

C-4\N-MATH\Ch1-1.pm5

Infinite Series

if,

if

if (1 1/n2)n > 1 1/n which is true by Bernoullis inequality. [For every positive integer n 2 and p > 1 (p 0) (1 + p)n > 1 + np]. Hence the given sequence is monotonic increasing. Example 3. Prove that the sequence un =

FG n 1IJ H n K FG n 1IJ H n K
2 2 2 2

>
n

n 1 n

> 1 1/n

i.e.,

4n + 3 4( n + 1) + 3 4n + 7 = , un+1 = n+2 ( n + 1) + 2 n+3 5 Now, un+1 un = >0 ( n + 2) ( n + 3) un+1 > un or, un < un+1 Putting n = 1, 2, 3, ...... we get u1 < u2 < u3 < ...... < un < ...... So {un} is monotonic increasing. 4n + 3 5 7 =4 < 4 and u1 = Now, un = n+2 n+2 3 7 < un < 4. Hence {un} is bounded. 3 Example 4. Discuss the convergence of the sequence {un} where 1 1 1 + + ...... + un = n +1 n + 2 n+n 1 1 1 1 + Now, un+1 un = = > 0 for all n. 2n + 1 2n + 2 n + 1 ( 2n + 1) ( 2n + 2) un+1 un for all n un < un+1 i.e., u1 < u2 < ...... < un < ...... So {un} is monotonic increasing. 1 1 1 1 , ......, is less than 1/n, it follows that un < n . = 1 for Also because each of n +1 n + 2 n+n n
{un} is bounded above. Hence, {un} is monotonic increasing and bounded above and is, therefore, convergent. VII. Infinite series. If {un} be a given real valued sequence, then an expression of the form u1 + u2 + ...... + un + ......
n =1

R 4n + 3 U is bounded and monotonic increasing. S n+2 V T W

all n.

is called an infinite series. In symbols it is generally written as

or

u.
n

VED

C-4\N-MATH\Ch1-1.pm5

Textbook of Engineering Mathematics

Whereas

n =1

is called a finite series assuming that the terms un+1, un+2, ...... are all zero.

VIII. Sequence of partial sums of an infinite series. Let us associate to the infinite series u1 + u2 + ...... + un + ...... a sequence {sn} defined by sn = u1 + u2 + ...... + un The sequence {sn} is called the sequence of partial sums of the given series. IX. Convergence of an infinite series. An infinite series un is said to converge, diverge or oscillate according as its sequence of partial sums {sn} converges, diverges or oscillates. In case {sn} converges to s, than s is called the sum of the series un and we shall write s = un. Mathematically, un converges if Lt sn = finite
n

un diverges if i.e.,

Lt sn = + or

un oscillates if {sn} oscillates (finitely/infinitely). {sn} does not have a unique limit. X. Series of positive terms. An infinite series in which all the terms are positive after some terms is called positive term series e.g., 2 1 + 1 + 2 + 3 + ...... is a positive term series.

1.2

GENERAL PROPERTIES OF SERIES


P1 P2 P3 P4 : Convergence of a series remains unchanged by the replacement, inclusion or omission of a finite number of terms. : A series remains convergent, divergent or oscillatory when each term of it is multiplied by a fixed number other than zero. : A series of positive terms either converges or diverges to + i.e., omitting the negative terms, the sum of first n terms tends to either a finite limit or + . : Every finite series is a convergent series.
n

Example 1. Show that if a series un is convergent then Lt un = 0. Let sn denote the nth partial sum of the series un. Then un is convergent {sn} is convergent. Lt sn = s (finite)
n

Also, But

Lt sn 1 = s
un = sn sn1
n

Lt un = Lt ( sn sn 1 ) = Lt sn Lt sn 1 = s s = 0
n n n n

The converse of the above result (Example 1) is not always true i.e., Lt un = 0, implies that un may or may not be convergent. 1 1 1 1 For example, 1 + + + ...... + + ...... diverges, though Lt un = Lt = 0. n n n n 2 3

VED

C-4\N-MATH\Ch1-1.pm5

Infinite Series

Example 2. The geometric series 1 + x + x2 + x3 + ...... (i) Converges if 1 < x < 1 (ii) Diverges if x 1 (iii) Oscillates finitely if x = 1 (iv) Oscillates infinitely if x < 1 (i) 1 < x < 1 Let i.e. | x | < 1 sn = 1 + x + x2 + ...... + xn1 xn 1(1 x n ) 1 = = 1 x 1 x 1 x 1 Lt sn = n 1 x {sn} is convergent, hence the given series converges

F3 H

Lt x n = 0 as | x | < 1

I K

(ii) x 1 i.e., x = 1 and x > 1 For x = 1, sn = 1 + 1 + 1 + ...... to n terms =n Lt sn =


n

{sn} is divergent hence the given series is also divergent For x > 1 sn = 1 + x + ...... + xn1

=
n

1 xn 1 x

Lt sn =

{sn} is divergent hence the given series is also divergent. (iii) x = 1 sn = 1 1 + 1 1 + 1 1 + ...... to n terms. = 1 or 0 if n is odd or even (iv) x < 1 Here, x < 1 Let Now
n

F3 H

Lt x n = , as x > 1

I K

Lt sn = 1 or 0.

{sn} oscillates finitely and hence the given series oscillates finitely. x>1 p = x, then p > 1 sn = 1 + x + x2 + ...... + xn1 1 x n 1 ( p) n = = 1 x 1+ p n 1 p 1 + pn = or if n = even or odd. 1+ p 1+ p

VED

C-4\N-MATH\Ch1-1.pm5

Textbook of Engineering Mathematics


n

Lt sn = or +

(3 pn as n )

{sn} oscillates infinitely and hence the given series oscillates infinitely.

1.3

COMPARISON TEST
I. If two positive term series un and vn be such that (i) vn converges and (ii) un vn n, then un also converges.
(Note. If the relation un vn holds for n > m, then the first m terms of both the series can be ignored which will not affect their convergence or divergence).

II. If two positive term series un and vn be such that (i) vn diverges and (ii) un vn n, then un also diverges. III. If two positive term series un and vn be such that Lt
n

un = l, l being a non-zero finite vn

quantity then un and vn either both convergent or both divergent. In particular, if l = 0 and vn converges, then un also converges, if l = and vn diverges, then un also diverges. Among I, II, III, III is widely used.
IV. An important series for comparison is the Harmonic series of order p i.e.

1
p

1 1 1 + ...... converges if p > 1 and diverges if p 1. p + p + p 1 2 3

Example 1. Test the convergence of the series 1 3 5 + + + ...... 1. 2 . 3 2 . 3 . 4 3 . 4 . 5 Tn of 1, 3, 5 = 2n 1, n = 1, 2, ...... 2n 1 2 1/n un = = 2 n( n + 1) ( n + 2) n (1 + 1/n) (1 + 2/n) 1 Let vn = 2 . Let us compare with un and vn n 2 1/n 2 un = Lt = Lt = 2 0 and finite n (1 + 1/n ) (1 + 2/n ) n v 1.1 n 1 1 is of the form with p = 2 > 1 3 vn = 2 np n vn is convergent un is convergent. Example 2. Test the convergence of the following series :

n2 + 1 n

j
n +1 n
2

un =

e =

n2 + 1 n

je

n2 + 1 + n

n2 + 1 + n

VED

C-4\N-MATH\Ch1-1.pm5

Infinite Series

1 n2 + 1 + n
1 . n

e
j

1 + 1/n 2 + 1

Let
n

vn =
Lt

un = Lt vn n

1 + 1/n 2 + 1

1 0 and finite 2

but

vn =

n diverges
1 3 2 + 2 4 3 + ...... 3

un also diverges. Example 3. Examine the convergence of the following series :


2 1 + 1

F 1 , p 1 for divergentI H n K
p

Here

un = =

n +1 n = n
n

d
i

n +1 n n
=
n n

n +1 + n

id

n +1 + n

i
3/ 2

1 n+1 + n

1 + 1/n + 1

1 + 1/n + 1

Let

vn =
n

1 n 3/ 2

Lt

un = Lt vn n

1 + 1/n + 1

1 2 0

and finite but vn is convergent un is also convergent.

F 1 , p > 1 for convergentI H n K


p

PROBLEMS
Test the Convergence or Divergence of the following series : 1. 3. 5. 7.
1 2 3 + + + ...... 1. 3 3 . 5 5. 7

2. 1 + 4.

1 2 2 33 44 + + + + ...... 22 33 4 4 55
2n3 + 5 4n5 + 1

FH n + 1 1 (n + n )
3
2 p

n n2 + 1

n3

IK

6. 8.

L MN

2 3 4 + + + ...... 1p 2 p 3 p
3

n3 + 1 n

OP Q

VED

C-4\N-MATH\Ch1-1.pm5

Textbook of Engineering Mathematics

ANSWERS
1. 5. 7. Divergent Convergent 2. Divergent 3. Convergent 8. Convergent. 4. Convergent 6. Convergent for p > 2 and divergent for p 2

Convergent for p > 1/2, divergent for p 1/2.

1.4

INTEGRAL TEST

A positive term series f(n) where f(n) = un decreases as n increases then the series un and the integral

for divergent the value of the integral must be infinite. Proof. Given that f(x) is a monotonic decreasing function. Let r be a positive number such that r + 1 x r 1. y f(r + 1) f(x) f(r) ur+1 f(x) ur Integrating in [r, r + 1]

f ( x ) dx converge or diverge together i.e., for convergent the value of the integral must be finite and

r +1

ur +1 dx
ur+1

0 1 r x r+1 n This inequality is true for the interval [r, r + 1]. Now consider the intervals [1, 2], [2, 3], ...... [r, r + 1], ......, [n 1, n]. The totality of all the intervals leads to the following inequality.

z z

r +1

r r +1

f ( x ) dx

f ( x ) dx ur

r +1

ur dx
...(i)

f(x)

u2 + u3 + ...... + un+1 i.e., sn+1 u1 Taking the limit n

(a) If

{sn} is a convergent sequence and un is convergent. (b) If


divergent sequence and un is divergent. Hence un and


Note. un and

z z

Lt sn +1 u1

z z z

f ( x ) dx +

n +1

f ( x ) dx sn

f ( x ) dx + ...... +

n +1

f ( x ) dx u1 + u2 + ...... + un.
(sn = un)

f ( x ) dx Lt sn

f ( x ) dx converges, then both

f ( x ) dx diverges, then both

z z
1

...(ii)

f ( x ) dx and Lt sn +1 u1 will be finite numbers. So


n

f ( x ) dx and Lt sn +1 u1 will be + . So {sn} is a


n

f ( x ) dx converge on diverge together.

f ( x ) dx , k 1 also converge or diverge together.

VED

C-4\N-MATH\Ch1-1.pm5

Infinite Series

Example 1. Show that the harmonic series of order p

n
1

1
p

1 1 1 ...... . + + 1p 2 p 3 p

Converges for p > 1 and diverges for 0 < p 1.

1 = f(n), for x 1, f(x) is positive, and monotonic decreasing. np Integral test is applicable.
Here un = The above series will converge or diverge according to Case (i) p 1

dx is finite or infinite. xp

dx = Lt xp k

= Case (ii) p = 1

1 (finite), for p > 1 = , for 0 < p < 1 p 1

k 1 p 1 dx Lt p = k 1 p x

FG H

IJ K

k dx dx = Lt ln k = = Lt k 1 x k 1 x un converges if p > 1 and diverges if 0 < p 1. Example 2. Test the convergence of the following series.

Here,

2n 3 n4 + 3
un =

2n 3 = f(n). n4 + 3 For x 1, f(x) is positive and monotonic decreasing. Integral test is applicable.
Now,

f ( x ) dx =

un is divergent.

k 2x3 2x 2 dx = Lt dx 4 4 k 1 x + 3 1 x +3 k 1 1 ln ( x 4 + 3) = Lt = Lt [ln ( k 4 + 3) ln 4] 1 k 2 k 2

PROBLEMS
Test the Convergence of the following series : 1.

1 n

2.

n ln n
1
2

VED

C-4\N-MATH\Ch1-1.pm5

10
3.

Textbook of Engineering Mathematics

1 n(ln n ) p

4.

n ln n ln (ln n)
1
3

5.

n ln n (ln (ln n))


1

6.

1 2 3 4 + + + + ...... 2 5 10 17 1 1 1 + 2 + 2 + ...... 3 1 5 1 7 1
2

7. 9.

1 + 2n + 3 1 2 3 + ...... 2 + 2 + 1+1 1+ 2 1 + 32
2

8. 10.

1 2 3 + + + ...... 23 33 43

ANSWERS
1. 2. 3. 7. Div. Div. Conv. 5. Conv. for p > 1, div. for p 1. 6. Div. 9. Div. 10. Conv. 4. Div. 8. Conv.

Conv. for p > 1, div. for p 1

1.5

DALEMBERTS RATIO TEST


n

If un be an positive term series and Lt then (i) un is convergent if m < 1. (ii) un is divergent if m > 1.

un +1 =m un

(iii) No conclusion can be drawn if m = 1 i.e., the test fails and the other method is required. Proof. (i) Let m > 1 and finite 1+ m < m, there exists a natural number l such that for 2 1+ m un > > 1 for all n l. 2 un+1 Taking vn =

FG 2 IJ H l + 1K

, we get

1+ m un v > n = for all n l 2 un + 1 v n + 1


Since vn is convergent, therefore, by comparison of ratios test, un is convergent. (ii) Let m < 1 and there exists a natural number l such that un < un+1 for all n l ul < un for all n l + 1
un < 1. un+1

VED

C-4\N-MATH\Ch1-1.pm5

Infinite Series

11

Lt un 0

un is divergent.
un = 1, but the first series n u n +1 is convergent and the second is divergent. So the series un may converge of diverge.

For m = 1, we note that both the series

and

n have
1

Lt

Hence the test fails for m = 1. Example 1. Discuss the convergent of the following series :

2 22 23 + ...... + 2 + 2 12 + 1 2 + 1 3 + 1 1 2 3 + + + ...... (ii) 2 1+ 2 1+ 2 1 + 23


(i) (i) Here un =

2n n +1
2

and un+1 =

2 n +1 ( n + 1) 2 + 1

Lt

un +1 2[ n 2 + 1] 2n 2 [1 + 1/n 2 ] = Lt = Lt 2 2 2 2 n [( n + 1) + 1] n n [(1 + 1/n ) + 1/n ] un


= Lt
n

2 [1 + 1/n 2 ] =2>1 (1 + 1/n) 2 + 1/n 2

Hence by ratio test, un is divergent. (ii) Here un =

n 1 + 2n

and un+1 =

n +1 1 + 2 n +1

Lt

1 2 n (2 n + 1) un + 1 ( n + 1) (1 + 2 n ) = Lt = Lt 1 + n n 2 n ( 2 n + 2) n un n(1 + 2 n +1 ) + 1) . + 2) = 1/2 < 1 un is convergent by ratio test.


= Lt
n n

F1 + 1 I . ( 2 H n K (2

F H

I K

Equivalent Statement If un be positive term series and Lt then (i) un is convergent is m > 1 (ii) un is divergent is m < 1 (iii) No conclusion can be drawn if m = 1. Example 2. Test for convergence for
n

un =m un + 1

1 2 1

x2 x4 x6 + + + ...... 3 2 4 3 5 4

VED

C-4\N-MATH\Ch1-1.pm5

12

Textbook of Engineering Mathematics

Here,

un =

x 2 n 2 ( n + 1) n

and un+1 =

Lt

un + 1 n +1 n = Lt n n + 2 n + 1 un
= Lt

FG H

1 + 1/n 1 . n 1 + 2/n 1 + 1/n

un converges according to x2 < 1 and diverges according to x2 > 1. 1 1 = 3/ 2 If x2 = 1, un = ( n + 1) n n (1 + 1/n) 1 Let vn = 3 / 2 n By comparison test,
n

FG H

IJ K

x 2n (n + 2) n + 1

1/ 2

x2

IJ K

1/ 2

. x2 = x2

Lt

1 un = Lt = 1 0 and finite. vn n 1 + 1/n

3 vn is convergent, un is also convergent. Hence, by ratio test the given series is convergent for x2 1 and is divergent for x2 > 1. Example 3. Test the convergence of the following series : (i)

Fn G2 H
1

2 n

1 , n2
2 2

IJ K

(ii)

xn (2 n ) !

(i) Here un =

( n + 1) 1 ( n + 1) 4 + 2 n +1 1 n + = n +1 + 2 , un+1 = 2 n +1 ( n + 1) 2 2 ( n + 1) 2 2n n
n

Lt

un + 1 [( n + 1) 4 + 2 n +1 ] n2 . 4 = Lt n 2 n +1 ( n + 1) 2 un n + 2n 1 . 2 . (1 + 1/n) 2 n 4 (1 + 1/n) 4 +

= Lt

LM N

2.2 n n4 n 4 [1 + 2 n /n 4 ]

OP Q

= Lt

1 (1 + 1/n) 4 + 2.2 n /n 4 1 . = .<1 2(1 + 1/n) 2 2 (1 + 2 n /n 4 )

Hence by the ratio test un converges. (ii) Here


Lt

un =

xn x n +1 un + 1 = ( 2n) ! (2 n + 2 ) !

1 (2n) ! u n +1 = Lt . x = Lt .x=0<1 n ( 2 n + 2 ) (2 n + 1) n (2n + 2) ! un

By DAlemberts ratio test, un converges.

VED

C-4\N-MATH\Ch1-1.pm5

Infinite Series

13

Example 4. Examine the convergence of the following series : 1 3! 5! 5! + + + + ...... 2 2.4 2.4.6 2.4.6.8 ( 2n 1) ! Here un = 2 . 4 . 6 . 8 ...... 2 n (2n + 1) ! un+1 = 2 . 4 . 6 . 8 ...... (2 n + 2) un + 1 (2n + 1) (2 n) (2n + 1) ! = Lt Lt = Lt n n u n (2 n 1) ! (2 n + 2) 2n + 2 n ( 2n + 1) . n ( 2 + 1/n) . n = Lt = Lt = n n (1 + 1/n ) n +1 By DAlemberts, ratio test the given series diverges.

PROBLEMS
Discuss the convergence of the following series : 1.

22 22 42 22 42 62 + ...... + . + . . 32 32 52 32 52 72
1 1 1 + + + ...... 1 1. 3 1. 3 . 5

2.

1 1. 3 1. 3 . 5 + + + ...... 4 4 . 7 4 . 7 . 10

3.

4.

n n +1
2

xn

5. 7. 8.

2p 3p 4p 2n . n ! + + 6. 1 + + ...... (p > 0) 2 ! 3! 4 ! nn + 1 ( + 1)(2 + 1) ( + 1)(2 + 1)(3 + 1) + + 1+ + ...... + 1 ( + 1)(2 + 1) ( + 1)( 2 + 1)(3 + 1)

n
1 + 3

n!
n

9.

n2 5n
3 32 33 + ...... + 2 + 3 2.3 2 .5 2 .7

5 2 . 32

10.

9 3 . 32

13 4 . 32
+ ...... 11. 1 +

12.

1 1 1 1 + + + ...... + + ...... 3 1+ e 1+ e 1+ e 1+ n e

ANSWERS
1. 4. 7. 10. Div. 2. Conv. 3. Conv. 5. Conv. 8. Conv. 12. Div. 6. Conv. 9. Conv. Conv. for x < 1 and div. for x 1 Conv. if > > 0, div. if > 0 Conv. 11. Div.

VED

C-4\N-MATH\Ch1-1.pm5

14 1.6

Textbook of Engineering Mathematics

RAABES TEST

The series un of positive terms is convergent or divergent according as


n

Lt

Rn F u | G S Hu | T

n+ 1

I U > 1 or < 1 V JK | | W

The test fails if the limit = 1. Generally, this test is applied when DAlemberts ratio test fails and also when in the ratio test

un +1 does not involve the number e. un


Proof. Let Lt (i) m > 1, for n Thus i.e., or

m +1 < m, there exists a natural number l such that 2

Rn F u |G S Hu | T
n n+1

n+1

IJ U = m be finite | KV | W

FG u Hu

1 >

IJ K

m +1 >1 2 m 1 un+1 for all n l, 2 m 1 2


n 1

nun (n + 1) un+1 >


n 1

[nun ( n + 1) un +1 ] >
lul >

u
l

n +1

m 1 (ul+1 + ...... + un) for all n l + 1, 2

2 lu + u1 + u2 + ...... + um > u1 + u2 + ..... + un for all n l + 1. m 1 l This shows that the sequence of partial sums {sn} is bounded and so, being monotonic it converges. Therefore un converges for finite m > 1.
(ii) If m < 1, there exists a natural number l such that n i.e.,
n 1

FG u Hu

1 < 1
nun < (n + 1)un+1 for all n l
n 1 n n +1

n+1

IJ K

i.e., i.e., i.e., Since


1

nu < (n + 1) u
l l

lul < nun for all n l + 1 1 (lul) . < un for all n l + 1 n

n is divergent and lul represents positive real number, un is divergent.

For m = 1, both the series

VED

C-4\N-MATH\Ch1-1.pm5

Infinite Series

15

n(ln n)
1
2

and

n have
1

Lt n

FG n Hu

1 = 1, but the first series is convergent whereas the

n +1

IJ K

second series is divergent. Hence for m = 1, the test fails.


Note. (De Morgans Test or Bertrands Test) Let un be a series of positive terms such that Lt if l > 1, (ii) un diverges if l < 1.

RLn F un 1I 1O log nU = l. Then (i) u converges |M G SM H un +1 JK PP | V n |N Q | T W


n

[This test may be applied when both DAlemberts ratio test and Raabes test fails].

Example 1. Discuss the convergence of the series : 1 1. 3 2 1. 3 . 5 3 x + x + ...... (x > 0) 1+ x+ 2 2.4 2.4.6 Neglecting the first term, we have 1 . 3 . 5 ...... ( 2n 1) n un = . x and 2 . 4 . 6 ...... 2n 1 . 3 . 5 ...... ( 2n 1) (2n + 1) n+1 un+1 = .x 2 . 4 . 6 ...... 2n . (2n + 2)

2n + 1 2 + 1/n un + 1 . x = Lt = Lt .x=x n u n 2n + 2 n 2 + 2/n n By DAlemberts ratio test, the series converges for x < 1 and diverges for x > 1. The test fails for x = 1. We apply Raabes test for x = 1. Lt
Now, n

Lt

By Raabes test the series diverges. Hence the given series converges for x < 1 and diverges for x 1. Example 2. Test the convergence of the series

FG u Hu Fu nG Hu

n+1 n

n +1

IJ FG 2n + 2 1IJ = n K H 2 n + 2 K 2n + 1 I F n IJ = Lt FG 1 IJ = 1 < 1 1J = Lt G H 2n + 1K H 2 + 1/n K 2 K


1 = n
n n

x 1 x3 1 . 3 x5 1 . 3 . 5 x7 + . + . + + ...... 1 2 3 2.4 5 2.4.6 7


un = Here, un+1 =

1 . 3 . 5 ...... ( 2n 3) x 2 n 1 . 2 . 4 . 6 ...... (2n 2) 2n 1 1 . 3 . 5 ...... ( 2n 1) x 2 n +1 . 2 . 4 . 6 ...... 2 n 2n + 1

un +1 (2n 1) 2 = . x2 2 n ( 2n + 1) un

VED

C-4\N-MATH\Ch1-2

16

Textbook of Engineering Mathematics

un +1 = x2 n u n by DAlemberts, ratio test, the series is convergent if x2 < 1, i.e., | x | < 1 and divergent, if 2 > 1, i.e., | x | > 1. x If x =1, the test fails. Let us apply Raabes test.
Now,

Lt

Here,

Lt

The series is convergent in this case. If x = 1, all the terms being reversed in sign as compared to the case when x = 1, the series remains convergent. Thus the given series is convergent when | x | 1, and divergent when | x | > 1.
Note. The series is the expansion of sin1 (x).

Ru Su T Ru nS Tu

1 =

n+1

n +1

U n(6n 1) = 6 1/n V (2n 1) (1 1/n) W U 6 3 1V = i.e., > 1 W 4 2


2

PROBLEMS
Test the convergence of the following series : 1.

2 . 5 . 8 ......(3n 1)
n =1

1 . 4 . 7 ...... (3n 2 )

2. 1 +

1 1. 3 1. 3. 5 + + + ...... 2.3 2. 4.5 2 .4 .6. 7

3.

1+

1 1. 3. 5 1. 3. 5 . 7 . 9 2.4 2.4.6 2.4.6.8 2 x+ + + + + ...... 4. x + ...... 2 . 4 2 . 4 . 6 . 8 2 . 4 . 6 . 8 . 10 . 12 3. 5 3. 5. 7 3. 5. 7. 9

5.

1 + 2x + 3x2 + 4x3 + ...... (x > 0)


22 22 . 42 22 . 42 . 62 x + 2 2 x 2 + 2 2 2 x3 + ...... 32 3 .5 3 .5 .7

6.

x x2 x3 x4 + ...... + + + 1. 2 3. 4 5 . 6 7 . 8 22 22 . 42 22 . 42 . 62 + ...... . + + 32 32 . 52 32 . 52 . 7 2

7.

1+

8. 1 +

ANSWERS
1. 4. 6. Div. Conv. if x 1, Div. if x > 1. 2. Conv. 7. Conv. if x < 1, Div. if x 1. 3. Conv. 8. Div. Conv. for x < 1 and Div. for x 1. 5. Conv. for x < 1, and Div, for x 1.

1.7

LOGARITHMIC TEST

A positive term series un converges or diverges according to


n

Lt

LMn log u OP > 1 N u Q


n n +1

or

<1

but the test fails for the limiting value = 1.

VED

C-4\N-MATH\Ch1-2

Infinite Series
Note. The test is applied after the failure of DAlembert ratio test and generally when This test is a substitute for Raabes test. Its proof is similar to that of Raabes test.

17

un involves e. un+1

Example 1. Test the convergence of the series x 2 ! 2 3! 3 + x + 2 x + ...... 2 32 4 ( n + 1) ! n+1 n! xn, un+1 = x Here, un = n ( n + 2) n +1 ( n + 1)
n

Lt

un + 1 ( n + 1) n +1 . x = Lt n +1 n (n + 2 ) un
= Lt

(1 + 1/n) n . (1 + 1/n) .x n n (1 + 2/n ) . (1 + 2/n)

e x = 2 .x= e e

LM Lt F1 + a I N H nK
n

= ea

OP Q

By DAlemberts ratio test, the series converges if i.e., x > e. If


x = 1 the test fails. e

x x < 1 i.e., x < e and diverges if >1 e e

Now when log

x = e,

un (1 + 2/n) n +1 1/e. = un +1 (1 + 1/n) n +1

un +1 = (n + 1) log (1 + 2/n) (n + 1) log (1 + 1/n) log e un

LMF 2 1 . 4 + 1 . 8 ......I F 1 1 . 1 + 1 ......I OP 1 KQ K H n 2 n 3n NH n 2 n 3 n 3 1 3 I = (n + 1) L 1 3 + ......O 1 = F 1 MN n 2n PQ H 2n + n 2n + ......K 1


= (n + 1)
2 3 2 3 3 2

= (n +1) [log (1 + 2/n) log (1 + 1/n)] 1

Lt n log

un u n +1

1 3 + ...... 2n 2n 2 1 3 = Lt n + ...... n 2n 2n 2

= Lt

F 1 3 + ......I = 1/2 < 1 H 2 2n K

F H

I K

So by Logarithmic test, the series diverges. Hence, the given series converges if x < e and diverges if x e.

VED

C-4\N-MATH\Ch1-2

18

Textbook of Engineering Mathematics

Example 2. Test the convergence of the series Here, un =

(1 + n) n n .x . n!

(1 + n) n n ( 2 + n) n +1 x , un+1 = . xn+1 n! ( n + 1) !

1 u Lt n +1 = Lt 1 + n u n n+1 n

By DAlemberts ratio test the given series converges or diverges according as x < 1/e or x > 1/e. For x = 1/e the test fails. n log

FG H

IJ K

n +1

. x = ex

un = n log un + 1

FG1 + 1 IJ H n + 1K L R F 2 I log F1 + 1 I UOP = n M1 (n + 1) Slog 1 + T H n K H n K VQ W N L RF I = n M1 (n + 1) |G 2 1 . 2 + ......J F 1 1 SH n 2 n K H n 2n | MN T


n +1 2 2

= n 1 (n 1) log 1 +

LM N

FG H

1 n+1

IJ OP KQ

+ ......

=
Lt n log

1 5 + ...... 2 6n

| I UOP K VPQ | W

1 un = < 1. Hence by the logarithmic test the series diverges for x = 1/e. Thus un +1 2 the given series converges or diverges according as x < 1/e or x 1/e.
n

PROBLEMS
Test the convergence of the following series : 1. 3. 5. 1+ 1+ 1+
2 32 43 + + .... 2 + 2 !. e 3 !. e 4 !. e 3 2x 32 x 2 43 x 3 54 x 4 e+ + + + .... 2! 3! 4! 5!

2. 1 + 4. x +

1 2! 3! 4! e + 2 e 2 + 3 e3 + 4 e 4 + .... 2 3 4 5
2 2 x 2 33 x 3 4 4 x 4 + + + .... 2! 3! 4!

1 2! 3! 4! x + 2 x 2 + 3 x 3 + 4 x 4 + ... 2 3 4 5

ANSWERS
1. 4. Conv. 2. Div.

Conv. if x < 1/e, Div. if x 1/e,

1 , Div. if x > 1/e e 5. Conv. if x < e, Div. if x e.


3. Conv. if x

VED

C-4\N-MATH\Ch1-2

Infinite Series

19

1.8

CAUCHYS ROOT TEST


n

If un is a positive term series and

Lt (un )1/ n = m, then the series is (i) convergent if m < 1 (ii)

divergent if m > 1 and no conclusion can be drawn about the convergence or divergence if m = 1. m +1 m +1 Proof. Let m 0. For m < < 1, there exists a natural number l such that 0 < un1/n < 2 2 n m +1 for all n l. Thus un < for all n l. 2
m +1 <1 2 By comparison test un is convergent. For m > 1, there exists, a natural number l such that un1/n > 1 un > 1 for all n l.
n

F H F m + 1I Since H 2 K

I K

is convergent for 0 <

Lt un 0

un diverges. 1/n + , there exists a natural number l. Such that n 1/n > l i.e., u > l for all n l If un n n
n

Lt un 0

un diverges

For m = 1, we note that,

1
2

and

n have
1

1/ Lt un n = 1 but the first series is convergent and

the second series is divergent. the test fails in this case. Example 1. Test the convergence of the following series

1+ 2 2+2 + 2 .1 2.2
Here
n

Lt

By Cauchys root test, the series is convergent. Example 2. Examine the convergence of the series

IJ + FG 3 + 2 IJ + ...... + FG n + 2 IJ K H 2.3 K H 2.n K F n + 2 IJ = F 1 + 1 I u =G H 2 . n K H 2 nK F 1 + 1I = 1 < 1 u = Lt H 2 nK 2 FG H


2 3 n n 1/ n n n

+ ......

[( n + 1) x ]n n n +1 [( n + 1) x ]n un = n n +1 ( n + 1) x ( n + 1) x 1 = Lt Lt un/ n = Lt 1/ n 1 + 1/ n n n . n n n n

VED

C-4\N-MATH\Ch1-2

20

Textbook of Engineering Mathematics

= Lt

By Cauchys root test, the series converges for x < 1 and diverges for x > 1.

F1 + 1 I . 1 H nK n

1/ n

.x=x

PROBLEMS
Test the convergence of the following series. 1.

3.

5.

F I + F 3 I + ...... H K H 7K F nI GH 1 + n JK 1 2 3 4 + x+F I x +F I H 4K H 5K 2 3
1 2 + 3 5
2 3 n2 2 2

F2 2. G H1

2 1

I +F3 JK GH 2
1 n

3 3

3 2

I JK

F4 +G H3

4 4

4 3

I JK

+ ......

4.
3

n=2

(log n)
1

x3 + ......

ANSWERS
1. Conv. 2. Conv. 3. Conv. 4. Conv.

5.

Conv. for x < 1, div. for x 1.

1.9

ALTERNATING SERIES : (LEIBNITZS TEST)

The alternating series

( 1)
1

n 1

un .
n

Converges if (i) un > un+1 for all n and (ii) Lt un = 0. Here both the conditions (i) and (ii) must be satisfied for convergence. If Lt un 0, the given series is oscillatory.
n

Proof. If Sn denotes the partial sum of the given alternating series then, S2n = u1 u2 + u3 u4 + ...... + u2n1 u2n = u1 [(u2 u3) + (u4 u5) + ...... + (u2n2 u2n1) + u2n] < u1 (3 un > un+1 for all n) This implies that {S2n} is bounded above Also, S2n+2 = S2n + u2n+1 u2n+2 S2n+2 S2n = u2n+1 u2n+2 S2n+2 S2n > 0 S2n+2 > S2n
Lt S2 n = finite

(3 un > un+1

u2n+1 > u2n+2 for all n)

This implies that {S2n} is monotonically increasing. Hence {S2n} is convergent i.e.,
n

VED

C-4\N-MATH\Ch1-2

Infinite Series

21

Also,

Lt S2 n +1 = Lt ( S2 n + u2 n +1 ) = Lt S2 n + 0
n n

F3 H

Lt un = 0 given

I K

= Lt S2 n = finite
n

Thus Lt Sn = finite when n is odd or even.


n

Hence the given series is convergent. When


n

Lt un 0,

Lt S2 n +1 Lt S2 n ,
n

The given series is oscillatory. Example 1. Discuss the convergence of the following series. 1

1 1 1 + + ...... 2 2 3 3 4 4

It is an alternating series. (i) un = Since,

1 n n

and un+1 =

1 ( n + 1) n + 1

1 n n

>

1 for all n, ( n + 1) n + 1

un > un+1 for all n (ii) Lt un = 0


n

Both the conditions of Leibnitzs test are satisfied. Hence the given series is convergent. Example 2. Test the convergence of the following series
1 2 3 4 5 + + ...... 6 11 16 21 26

It is an alternating series (i) 3 (ii)


n

un =

n n +1 , un+1 = 5n + 1 5(n + 1) + 1

n n +1 > for all n. 5n + 1 5(n + 1) + 1


un > un+1 for all n.

Lt un = Lt

1 n 1 = Lt = 0 n 5 + 1/n 5n + 1 5

Second condition of Leibnitzs test is not satisfied. Hence the given series is Oscillatory.

VED

C-4\N-MATH\Ch1-2

22

Textbook of Engineering Mathematics

PROBLEMS
1. Show that the series

F 3 4 I + F 5 6 I + ...... + FG 2n + 1 2n + 2 IJ H 2 3 K H 4 5K H 2n 2n + 1 K
3 4 5 6 2n + 1 2n + 2 + + ...... + + ...... 2 3 4 5 2n 2n + 1

+ ......

is convergent but the series

obtained from the first by omitting brackets is not convergent. 2. Show that the series 1
1 1. 3 1. 3 . 5 + + ...... 2 2.4 2.4.6

is conditionally convergent. [Hints. See the next section] 3. Examine the convergence of the following series : (i) 1

1 1 1 1 + + + ...... 2 3 4 5

(ii)

M N
1

L ( 1)
n

n 1

2n 1 sin

OP Q
(1/n) .

1 1 1 1 + (iii) + ...... log 2 log 3 log 4 log 5

(iv)

( 1)

ANSWERS
3. (i) Conv. (ii) Oscillatory (iii) Conv. (iv) Conv.

1.10 SERIES OF POSITIVE AND NEGATIVE TERMS


The series of Positive terms and alternating series are the special types of these series with arbitrary signs. The series un of arbitrary terms is said to be absolutely convergent if | un | is a convergent series. Whereas, the series un is said to be conditionally convergent if it is convergent but does not converge absolutely i.e., | un | is divergent. To test | un | is convergent or not, suitable tests as discussed in the earlier sections has to be applied. Example 1. Test whether the series is absolutely convergent or conditionally convergent ? 1 This is an alternating series (i) un =
1 n2 1 1 1 + ...... 2 + 2 2 2 3 4

and un+1 =

1 obviously, un > un+1 for all n. ( n + 1) 2

VED

C-4\N-MATH\Ch1-2

Infinite Series

23

(ii) Lt un = 0.
n

Both the conditions of the Leibnitzs test are satisfied, so the given series is convergent. Again, consider the series with absolute terms 1 1 1 1 | un | = 1 + 2 + 2 + 2 + ...... = n2 2 3 4 Which is a convergent series, (p = 2 > 1 in the Harmonic Series). Thus the given series converges absolutely. Example 2. Prove that the following series converges absolutely.

x3 x5 x7 + ...... + + 3! 5! 7! Consider the series by taking all the terms positive


x i.e., Here,

x+
un =

x3 x5 x7 + + + ...... 3! 5! 7!

...(i)

x 2 n +1 x 2 n 1 , un+1 = ( 2n + 1) ! ( 2n 1) !

un +1 x2 = Lt = 0 for all x. n u n (2 n + 1) 2 n n Hence the series (i) is convergent for all x. Therefore the given series is absolutely convergent for all x. Result. Every absolutely convergent series is convergent but the converse is not true. Let un be an absolutely convergent series. Obviously, u1 + u2 + ...... + un + | u1 | + | u2 | + ...... + | un | + ...... i.e., un | un | where | un | is convergent. Hence un is also convergent. Lt

PROBLEMS
1. 2. Text the convergence of the series 1 +

1 1 1 1 1 1 1 1 + + + + + ...... . 2 3 4 5 6 7 8 9

If S is the sum of the conditionally convergent series

3.

( 1)n 1 , then show that the rearranged series. n 1 1 1 1 1 1 1 1 + + ...... has the sum = S. 2 2 4 3 6 8 5 Test whether the following series are absolutely convergent or conditionally convergent ?

(i) (iii)

( 1)n 1 2n 1
( 1)n 1 n n2 + 1

(ii) 1 +

1 1 1 1 1 1 1 2 2 2 + 2 + 2 2 2 + ...... 2 3 4 5 6 7 8
5

(iv) 1 +

1 1 1 + ...... 2 53 54

VED

C-4\N-MATH\Ch1-2

24

Textbook of Engineering Mathematics

(v) 4.

n log
2

( 1)n1 2 n

(vi)

( 1)n . 23n 32 n

Prove that the series

sin x sin 2 x sin 3x + ...... 13 23 33


Converges absolutely. 5. Show that the series
1 1 1 1 1 + + ...... + ( 1)n + ...... log 2 log 3 log 4 log 5 log n Converges conditionally.

ANSWERS
1. 3. Div. (i) Converges conditionally, (iii) Conditionally conv. (v) Conditionally conv. (ii) Absolute conv. (iv) Conditionally conv. (vi) Absolute conv.

1.11 POWER SERIES


A series of the form a0 + a1x + a2 x2 + ...... + an xn + ...... where a0, a1, ...... an, ...... are all constants is called a power series in x which may converge for some or all values of x. These values form the interval of convergence. Also every power series is convergent for x = 0. Example 1. Show that the exponential series 1+x+ Here
Lt

x2 xn + ...... is convergent for all values of x. + ...... + 2! n!


un =

xn x n+1 , un+1 = ( n + 1) ! n!

u n +1 x = Lt =0 n u n n n Hence by DAlemberts ratio test the series converges for all values of x.

Example 2. Show that the logarithmic series x Here,


n x2 x3 n x + ...... is convergent for 1 < x 1. + ...... + 1 2 3 n x n+1 xn , un+1 = ( 1)n+1 . un = ( 1)n . n +1 n

a f FG H

Lt

1 un +1 n = x Lt = x Lt n n + 1 n 1 + 1/n un

The series converges for | x | < 1 and divergent for | x | > 1, when x = 1, the series 1 1 1 1 + + ...... is convegent by Leibnitzs test. 2 3 4

IJ K

FG H

IJ = x K

VED

C-4\N-MATH\Ch1-2

Infinite Series

25

When x = 1, the series 1 1 1 1 + + + + ...... is divergent (comparing with Harmonic series) 2 3 4 Hence the series converges for 1 < x 1. Example 3. For what values of x the following series is convergent ? x + 2 ! x 2 + 3 ! x 3 + ...... + n ! x n + ...... 22 33 nn (n + 1) ! n+1 n! Here un = n xn, un+1 = x ( n + 1) n +1 n

u ( n + 1) ! n n n Lt n +1 = Lt . x = Lt n +1 n n +1 n ( n + 1) n u n! n
= Lt
n

FG H

IJ K

.x

1 x n . x= (1 + 1/n) e |x| For convergence by DAlemberts ratio test < 1 | x | < e i.e., e < x < e. e

PROBLEMS
For what values of x are the following series convergent : 1. 2. 3. 5. 6. 8. 10. x
x2 x3 x4 + + ...... 2 3 4 n( n 1) 2 n(n 1) ...... (n r + 1) r x + ...... + x + ...... 1 + nx + r! 2! x x2 xn x 2 x 3 x 4 x 5 + ...... 4. + ...... + + ...... + + n+ n 1+ 1 2 + 2 2 3 4 5 x x x + 4 sin + ...... + 2n sin n + ...... sin x + 2 sin 3 9 3

x 1+

x3 x5 ...... + 3 5

3 1 ( x 3)2 1 ( x 3)3 + ...... 7. x + + 3 2 3 33 32


9. 1 ! . x + 2 ! x2 + 3 ! x3 + ...... + n ! xn + ......

x2 x 4 x6 + + + ...... 2! 4! 6! 1 2 1. 3 4 1. 3. 5 6 x + 1+ x + x + ...... 2 2.4 2.4.6

ANSWERS
1. 4. 7. 10. 1<x1 1x<1 0x<6 1 < x < 1. 2. 1 < x < 1 5. < x < 8. < x < 3. 1 < x 1 6. 1 x 1 9. x = 0

VED

C-4\N-MATH\Ch1-2

26

Textbook of Engineering Mathematics

1.12 UNIFORM CONVERGENCE AND WEIERSTRASSS M-TEST


Let Let and

u ( x ) be an infinite series of functions each of which is defined in the interval (a, b).
n 1

sn(x) = u1(x) + u2(x) + ...... + un(x) = sum of first n terms s(x) = Lt sn ( x )


n

The series un(x) is said to be uniformly convergent in the interval (a, b), if for a given > 0, there exists a number N which is independent of x, such that for every x in the interval (a, b), | s(x) sn(x) | < for all n > N By definition, uniform convergence implies convergence for each point x in (a, b) but the converse may or may not hold. Example 1. The series x x x + + + ...... x + 1 ( x + 1) (2 x + 1) (2 x + 1) (3 x + 1) Converges uniformly on [a, ) where a > 0. Here and so,

x x x 1 + + ...... + =1 x + 1 ( x + 1) (2 x + 1) (( n 1) x + 1) ( nx + 1) nx + 1 1 Rn(x) = nx + 1
sn(x) =

Let 0 < < 1. Then | Rn(x) | < for n >

rn(x) | < for all n N and x a > 0. Hence the same N serves for the convergence of the series for all x is [a, ) and so the given series converges uniformly on [a, ), a > 0. Weierstrasss M-Test for Uniform Convergence A series un (x) is uniformly convergent in an internal (a, b), if there exists a convergent series Mn of non-negative terms such that | un(x) | Mn for all values of x in (a, b). Proof. If Mn is convergent, for > 0, there exists, N such that | Mn+1 + Mn+2 + ...... + Mn+p | < for all n N and p 1. For all x in (a, b)
|un+ 1 ( x) + un+ 2 ( x) + ...... + un+ p ( x)| |un+ 1 ( x)|+|un+ 2 ( x)| + ...... +|un+ p ( x)|

F 1 1I H K

x . Thus if we take N

F 1 1I H K

a , we get |

| Mn+ 1 |+| Mn + 2 |+ ...... +| Mn+ p | | Mn+ 1 + Mn+ 2 + ...... + Mn+ p |

< for n N and p 1 Hence, un(x) converges uniformly on (a, b).

VED

C-4\N-MATH\Ch1-2

Infinite Series

27

Also from above, | un+1 (x) | + | un+2 (x) | + ...... + | un+p (x) | < for all n N and p 1 on (a, b). Therefore, un(x) also converges absolutely on (a, b). Example 2. Show that the series

n =1

x 1 + n2 x2

Converges uniformly on [a, 1], 0 < a < 1 but not on [0, 1]. On [a, 1] where 0 < a < 1,
| un ( x ) | = 1 x 1 + n2 x2 1 + n2 a 2

and now, to test the convergence of Let


Lt

1 1 + n 2 a2

fn =

1 1 , f n +1 = 2 2 1+ n a 1 + ( n + 1) 2 a 2

Now, we shall apply Raabes test

1 + n2 a 2 a2 1/n 2 + a 2 fn +1 = Lt = Lt = 2 =1 2 2 2 2 2 n 1 + ( n + 1) a n f n 1/n + (1 + n/1) a a n DAlemberts ratio test fails

fn converges Therefore by Weierstrasss M-Test, the given series converges uniformly on [a, 1] On the otherhand, let the given series uniformly convergent on [0, 1], then for = exists m, such that
1 > 0 there 8

Ff nG Hf Ff Lt n G Hf

n+1

n +1

I F 1 + (n + 1) a 1IJ = n FG 2na + a IJ 1J = n . G K H 1+ n a K K H 1+ n a I n . n (2 a + a /n) 2 a 1J = Lt = = 2 >1 n (1/n + a ) K a


2 2 2 2 2 2 2 2 2 2 2 n 2 2 2 2

x x x 1 + + ...... + < 2 2 2 2 2 2 1+ m x 1 + ( m + 1) x 1 + (2 m ) x 8

mx 1 < 1 + (2 m ) 2 x 2 8

(by taking n = m)

On putting x = convergent on [0, 1].

1 1 1 it gives < , a contradiction, therefore the given series is not uniformly m 5 8

VED

C-4\N-MATH\Ch1-2

28

Textbook of Engineering Mathematics

Example 3. Show that the series 3n sin

1 converges absolutely and uniformly on (a, ) 4n . x

where a > 0. For any x > 0 in (a, ), there exists a N such that 4n x 0 for all n N. Hence the series after a finite number of terms consists of positive terms.
3 un +1 4 .x = Lt 3 . = <1 1 4 n n un sin n 4 .x The series converges absolutely on (a, ) if a > 0. 1 1 1 Also, for n M, sin n < n < n N . 4 x 4 x 4

sin

1
n +1

Since

Lt

n Thus, 3 sin

1 4n x
n

< 4N

F 3I H 4K

for all n N

Since 4N

uniformly on (a, ) where a > 0.

F 3I H 4K

is convergent, therefore, by Weierstrasss M-Test, 3n sin 1/4n x converges

PROBLEMS
1.
x has a maximum at x = n(1 + nx 2 ) converges uniformly in (0, ). Test for uniform convergence of the series

Show that

1 and hence or otherwise show that n

n(1 + nx )
x
2

2.

(i) (iii)

n
n =1

nx 2 on [0, k] for any k > 0 + x3

(ii) sin x (iv)

sin 2 x sin 3x sin 4 x + + ... 2 2 3 3 4 4

cos nx and 2n

sin nx 2n

x n(1 + nx 2 )
x2 x 4 x6 + ... + 2! 4! 6!

(v) x

x3 x 5 x 7 + ... + 3! 5! 7!

(vi) 1

ANSWERS
2. (i) Uniform conv. on [0, k], k > 0 (ii) (vi) Uniform conv. for any real x.

REVIEW QUESTIONS
1. 2. Give the difference between sequence and series. Give an example of a monotonically increasing sequence which is convergent.

VED

C-4\N-MATH\Ch1-2

Infinite Series
3. 4. 5. 6. 7. 8. 9. What is meant by Oscillatory sequence. State the properties of series. What is meant by convergent of a series. Under what condition, Logarithmic test is applied instead of Raabes test. Define uniform convergence of a series. State Weierstrass M-Test for Uniform Convergence of a series. Define absolute and conditional convergence with examples.

29

OBJECTIVE QUESTIONS
1. For the sequence .6, .66, .666, ... (a) 0 2. (a) 0 3. (b) 1 (b) 1
2 1 1 n ... the least upper bound is 3 10 (c) 2/3
n1

F H

I K

(d) 1/3 (d)

For the sequence 1, 1, 1, 1, ... ( 1)

, ..., the greatest lower bound is (c) 1

4. 5.

3 2 5 4 7 6 , , , , , ... is 2 3 4 5 6 7 (a) 0 (b) 1 (c) 2 n}, the limit points are For the sequence {( 1)
The limit point of the sequence 0, (a) 0, 1 (a) Converges (c) Oscillates finitely (b) 0, 1 x2 + x3 (c) 1, 1 + ... . (b) Diverges (d) Oscillates infinitely For x = 1, the geometric series 1 + x +

(d) Does not exist (d) Does not exist

6.

If m is a given positive integer, then consider two series, S1 = u1 + u2 + ... + um+1 + un+2 + ... and S2 = um+1 + um+2 + ... (b) S1 Converges, S2 Converges (d) S1 Oscillates, S2 Diverges (a) S1 Converges, S2 Diverges (c) S1 Diverges, S2 Converges

7.

The series (a) p = 1

1
p

converges for (b) p 1 (c) p > 1


un +1 involves e then un

(d) p 1

8.

When D Alemberts ratio test fails and (a) Raabes test is applied (c) Logarithmic test is applied

(b) Cauchys Root test is applied (d) Integral test is applied (b) D Alemberts ratio test is applied (d) Integral test is applied (b) Conditionally converges (d) Uniformly converges

9.

On Alternating series ( 1)n1 un, (a) Cauchys Root test is applied (c) Leibnitzs test is applied

10.

In the interval [ 1/2, 1/2], the geometric series 1 + x + x2 + x3 + ... (a) Converges (c) Absolutely converges

VED

C-4\N-MATH\Ch1-2

30
11.

Textbook of Engineering Mathematics


If Lt
an +1 = l (finite), then the interval of convergence for the power series a0 + a1x + ... + anxn + ... is an

(a) 0 < x < l (c) 12. 13.

(b) 0 < x < 1/l

1 1 <x< (d) l < x < l l l Every power series of the form a0 + a1x + ... + anxn + ... is convergent for
(a) x = 0 (a) x < 0 (b) x = 1 (b) x > 1 (c) x < 1 (c) 0 < x < 1 (d) x 1 (d) 1 < x < 1 The series 1 x + x2 x3 + ... converges absolutely if

14.

The series 1

1 1 1 + + ... is 2 3 4 (a) Uniformly convergent


(c) Conditionally convergent

(b) Absolutely convergent (d) Divergent

15.

The series (a) x < 0

x is convergent if 2 n +1

(b) 0 < x < 1

(c) x 1

(d) x 1.

ANSWERS
1. 5. 9. 13. (c) (c) (c) (c) 2. (c) 6. (b) 10. (d) 14. (c) 3. (b) 7. (c) 11. (c) 15. (c). 4. (c) 8. (c) 12. (a)

VED

C-4\N-MATH\Ch1-2

You might also like